Solving Limit to inf Problem: Use Indeterminate Forms & l'Hospital's Rule

  • Thread starter chocolatedady
  • Start date
  • Tags
    Limit
In summary, the conversation is about someone seeking help with a limit question and thinking that using indeterminate forms and l'hospital's rule would be the solution. They are struggling with setting it up and are seeking assistance. The conversation also includes a suggested function and a thank you for help with the answer.
  • #1
chocolatedady
2
0
Hi, I have a limit question which I can't find out the answer to.
I think using indeterminate forms and l'hospital's rule is the solution in doing this but can't find out the right way to set it up.

[PLAIN]http://img130.imageshack.us/img130/662/lim.png

Any help would be appreciated.
 
Last edited by a moderator:
Physics news on Phys.org
  • #2
chocolatedady said:
Hi, I have a limit question which I can't find out the answer to.
I think using indeterminate forms and l'hospital's rule is the solution in doing this but can't find out the right way to set it up.

[PLAIN]http://img130.imageshack.us/img130/662/lim.png

Any help would be appreciated.

What if you write your function as follows?

[tex]{{(1-5/x)^{1/3}-1}\over{1/x}}[/tex]
 
Last edited by a moderator:
  • #3
Awesome, thanks a lot man! -5/3 for the answer
 

What is a limit to infinity problem?

A limit to infinity problem is a type of mathematical problem that involves finding the value of a limit as the input variable approaches infinity. It is often used to determine the behavior of a function as the input variable becomes infinitely large.

What are indeterminate forms?

Indeterminate forms are mathematical expressions that cannot be evaluated using standard algebraic techniques. They often arise in limit problems and require special methods, such as l'Hospital's rule, to solve them.

What is l'Hospital's rule?

L'Hospital's rule is a mathematical technique used to evaluate limits involving indeterminate forms. It states that for certain types of indeterminate forms, the limit of the quotient of two functions is equal to the limit of the quotient of their derivatives.

How do you solve a limit to infinity problem using indeterminate forms?

To solve a limit to infinity problem using indeterminate forms, you first need to identify the indeterminate form in the expression. Then, you can apply l'Hospital's rule to evaluate the limit. If the indeterminate form is not in the correct form for l'Hospital's rule, you may need to manipulate the expression algebraically before applying the rule.

What are some common mistakes when using l'Hospital's rule to solve limit to infinity problems?

Some common mistakes when using l'Hospital's rule include forgetting to check if the limit is actually indeterminate, using the rule on a limit that is not in the correct form, and incorrectly applying the rule multiple times. It is important to carefully analyze the problem and follow the steps of l'Hospital's rule correctly to avoid these mistakes.

Similar threads

  • Calculus and Beyond Homework Help
Replies
4
Views
969
  • Calculus and Beyond Homework Help
Replies
8
Views
1K
Replies
35
Views
3K
  • Calculus and Beyond Homework Help
Replies
5
Views
1K
Replies
4
Views
2K
  • Calculus and Beyond Homework Help
Replies
14
Views
1K
  • Calculus and Beyond Homework Help
Replies
6
Views
1K
  • Calculus and Beyond Homework Help
Replies
4
Views
1K
  • Calculus and Beyond Homework Help
Replies
15
Views
1K
  • Calculus and Beyond Homework Help
Replies
3
Views
1K
Back
Top